256

Let $U$ be an open set in $\mathbb R$. Then $U$ is a countable union of disjoint intervals.

This question has probably been asked. However, I am not interested in just getting the answer to it. Rather, I am interested in collecting as many different proofs of it which are as diverse as possible. A professor told me that there are many. So, I invite everyone who has seen proofs of this fact to share them with the community. I think it is a result worth knowing how to prove in many different ways and having a post that combines as many of them as possible will, no doubt, be quite useful. After two days, I will place a bounty on this question to attract as many people as possible. Of course, any comments, corrections, suggestions, links to papers/notes etc. are more than welcome.

Arctic Char
  • 13,467
  • 17
  • 21
  • 45
Orest Xherija
  • 1,039
  • 3
  • 12
  • 26
  • 108
    First proof that comes in mind: if $O$ is an open set and $x \in O$ then there exists an interval $I$ such that $x \in I \subset O$. If there exists one such interval, then there exists one 'largest' interval which contains $x$ (the union of all such intervals). Denote by $\{I_\alpha\}$ the family of all such maximal intervals. First all intervals $I_\alpha$ are pairwise disjoint (otherwise they wouldn't be maximal) and every interval contains a rational number, and therefore there can only be a countable number of intervals in the family. – Beni Bogosel Mar 02 '13 at 00:12
  • 1
    Since this is a [tag:big-list] question, I am converting it to CW. – robjohn Mar 02 '13 at 09:01
  • Oh, OK! Thanks! – Orest Xherija Mar 03 '13 at 01:04
  • I cannot understand how each $I_a$ is disjoint. If one interval contains x and another interval also contains x, aren't they intersecting? – mesllo May 29 '14 at 01:41
  • Yes but their union would then be another interval that contains an $I_a$ and therefore must be equal to it by maximality. – Gregory Grant Mar 15 '15 at 16:36
  • @GregoryGrant: what does α mean? Where does α come from? – Bear and bunny Mar 27 '15 at 14:12
  • It's an index variable on the family $\{I_\alpha\}$. – Gregory Grant Mar 27 '15 at 14:19
  • @GregoryGrant: I cannot understand why it is disjoint here. Is {α} equal to the open set $O$? – Bear and bunny Mar 27 '15 at 14:32
  • @GregoryGrant: I can understand "union would then be another interval that contains an $I_α$ and therefore must be equal to it by maximality". However, two $I_α$ may contain elements from the open set $O$ except x. – Bear and bunny Mar 27 '15 at 14:40
  • @GregoryGrant: Ohhh. It seems I got your idea. {α} is a proper subset of the open set $O$. – Bear and bunny Mar 27 '15 at 15:21
  • @OrestXherija: any $\sigma$-compact space is Lindelöf. See for instance https://proofwiki.org/wiki/Sigma-Compact_Space_is_Lindel%C3%B6f. Any separable metric space is Lindelöf, see e.g. http://math.stackexchange.com/questions/621180. These properties can be useful to address your question. We just have to pay attention to have _disjoint_ open sets at the end. – Watson Aug 23 '16 at 12:41
  • No one mentions it's UNIQUE DECOMPOSITION! Supposing no "empty intervals" in the union, the intervals in one decomposition must be the same as the other. – Santropedro Mar 16 '17 at 02:58
  • It's reminiscent of an exercise in baby Rudin... – YuiTo Cheng Dec 15 '18 at 09:07
  • @YuiToCheng It would not surprise me. I think I might have seen some version of it in Munkres' Topology. – Orest Xherija Jun 28 '19 at 04:57

16 Answers16

176

Here’s one to get things started.

Let $U$ be a non-empty open subset of $\Bbb R$. For $x,y\in U$ define $x\sim y$ iff $\big[\min\{x,y\},\max\{x,y\}\big]\subseteq U$. It’s easily checked that $\sim$ is an equivalence relation on $U$ whose equivalence classes are pairwise disjoint open intervals in $\Bbb R$. (The term interval here includes unbounded intervals, i.e., rays.) Let $\mathscr{I}$ be the set of $\sim$-classes. Clearly $U=\bigcup_{I \in \mathscr{I}} I$. For each $I\in\mathscr{I}$ choose a rational $q_I\in I$; the map $\mathscr{I}\to\Bbb Q:I\mapsto q_I$ is injective, so $\mathscr{I}$ is countable.

A variant of the same basic idea is to let $\mathscr{I}$ be the set of open intervals that are subsets of $U$. For $I,J\in\mathscr{I}$ define $I\sim J$ iff there are $I_0=I,I_1,\dots,I_n=J\in\mathscr{I}$ such that $I_k\cap I_{k+1}\ne\varnothing$ for $k=0,\dots,n-1$. Then $\sim$ is an equivalence relation on $\mathscr{I}$. For $I\in\mathscr{I}$ let $[I]$ be the $\sim$-class of $I$. Then $\left\{\bigcup[I]:I\in\mathscr{I}\right\}$ is a decomposition of $U$ into pairwise disjoint open intervals.

Both of these arguments generalize to any LOTS (= Linearly Ordered Topological Space), i.e., any linearly ordered set $\langle X,\le\rangle$ with the topology generated by the subbase of open rays $(\leftarrow,x)$ and $(x,\to)$: if $U$ is a non-empty open subset of $X$, then $U$ is the union of a family of pairwise disjoint open intervals. In general the family need not be countable, of course.

Zhanxiong
  • 12,018
  • 1
  • 21
  • 47
Brian M. Scott
  • 588,383
  • 52
  • 703
  • 1,170
  • 22
    I like this answer LOTS. :-) And +1. – coffeemath Mar 02 '13 at 01:30
  • 2
    This answer is very clear. It depends on the axiom of choice though, are there any constructive variants of this argument? – Bunder Mar 02 '13 at 08:28
  • 8
    @Bunder: No, it doesn’t depend on the axiom of choice. The only place where you might even think that it did is when I showed that there are only countably many intervals, but the rationals can be explicitly well-ordered, so no choice is necessary even there. – Brian M. Scott Mar 02 '13 at 12:24
  • Is it correct to write each equivalence class as $I_x = (\inf\{y \in U: y \sim x\}, \sup\{y \in U: y \sim x\})$? – David Nov 04 '15 at 16:09
  • @David: Yes, provided that you allow $\infty$ and $-\infty$ as possible values. – Brian M. Scott Nov 04 '15 at 16:54
  • Can this be possible in $R^n$ ?@BrianM.Scott – anonymous Mar 20 '18 at 12:59
  • 1
    Why are the equivalence classes open? – IntegrateThis Sep 20 '18 at 22:14
  • 3
    @IntegrateThis. Let $S$ be a $\sim$-equivalence class. For any $x\in S$ there exists $r>0$ such that $(-r+x,r+x)\subset U$ so $\forall y\in (-r+x,r+x)\;(y\sim x). $ So $(-r+x,r+x)\subset \{y\in U:y\sim x\}=S.$ – DanielWainfleet Sep 24 '18 at 20:38
  • Brilliant ok! @DanielWainfleet – IntegrateThis Oct 27 '18 at 00:08
  • 1
    This is very nice! I have assigned this problem as an exercise in more than course, and I always thought that the crux of it was that the connected subsets of the real line are precisely the intervals (a property that, among densely linearly ordered spaces, is equivalent to Dedekind completeness). Your answer shows that this is not the case at all. – Pete L. Clark Apr 30 '21 at 22:54
65

These answers all seem to be variations on one another, but I've found each one so far to be at least a little cryptic. Here's my version/adaptation.

Let $U \subseteq \mathbb{R}$ be open and let $x \in U$. Either $x$ is rational or irrational. If $x$ is rational, define \begin{align}I_x = \bigcup\limits_{\substack{I\text{ an open interval} \\ x~\in~I~\subseteq~U}} I,\end{align} which, as a union of non-disjoint open intervals (each $I$ contains $x$), is an open interval subset to $U$. If $x$ is irrational, by openness of $U$ there is $\varepsilon > 0$ such that $(x - \varepsilon, x + \varepsilon) \subseteq U$, and there exists rational $y \in (x - \varepsilon, x + \varepsilon) \subseteq I_y$ (by the definition of $I_y$). Hence $x \in I_y$. So any $x \in U$ is in $I_q$ for some $q \in U \cap \mathbb{Q}$, and so \begin{align}U \subseteq \bigcup\limits_{q~\in~U \cap~\mathbb{Q}} I_q.\end{align} But $I_q \subseteq U$ for each $q \in U \cap \mathbb{Q}$; thus \begin{align}U = \bigcup\limits_{q~\in~U \cap~\mathbb{Q}} I_q, \end{align} which is a countable union of open intervals.

Yia War
  • 15
  • 5
Stromael
  • 1,312
  • 1
  • 11
  • 22
  • 1
    He wanted a countable union of disjoint open intervals. If $U$ is simply an open interval, then $U=I_q=I_{\tilde{q}}$ for all $q,\tilde{q} \in \mathcal{Q}\cap U$. – José Siqueira Oct 15 '13 at 16:15
  • 9
    That it is a disjoint union follows from the definition of $I_q$: if $x \in I_q \cap I_p$ then $I_q \cup I_p \subseteq I_q$ and $I_p$; hence if $I_q \neq I_p$ then $I_q \cap I_p = \emptyset$. Strictly speaking one should throw away all repeated $I_q$s, and one can definitely do this without destroying the countability of the union. – Stromael Oct 21 '13 at 11:14
22

In a locally connected space $X$, all connected components of open sets are open. This is in fact equivalent to being locally connected.

Proof: (one direction) let $O$ be an open subset of a locally connected space $X$. Let $C$ be a component of $O$ (as a (sub)space in its own right). Let $x \in C$. Then let $U_x$ be a connected neighbourhood of $x$ in $X$ such that $U_x \subset O$, which can be done as $O$ is open and the connected neighbourhoods form a local base. Then $U_x,C \subset O$ are both connected and intersect (in $x$) so their union $U_x \cup C \subset O$ is a connected subset of $O$ containing $x$, so by maximality of components $U_x \cup C \subset C$. But then $U_x$ witnesses that $x$ is an interior point of $C$, and this shows all points of $C$ are interior points, hence $C$ is open (in either $X$ or $O$, that's equivalent).

Now $\mathbb{R}$ is locally connected (open intervals form a local base of connected sets) and so every open set if a disjoint union of its components, which are open connected subsets of $\mathbb{R}$, hence are open intervals (potentially of infinite "length", i.e. segments). That there are countably many of them at most, follows from the already given "rational in every interval" argument.

Henno Brandsma
  • 227,713
  • 7
  • 90
  • 220
  • "connected => interval" also requires a proof (usually using the intermediate step "path-connected"). – Martin Brandenburg Mar 04 '13 at 19:29
  • 1
    Path connected is not needed. It would be circular, as one needs connectedness of intervals to see that path-connected implies connected... If a set is not order convex (so x < z < y, x,y in A but z not in A), then we have an immediate disconnection. That intervals are connected follows from completeness of the order, and is a standard fact for ordered spaces. – Henno Brandsma Mar 04 '13 at 20:22
20

Let $U\subseteq\mathbb R$ open. It is enough to write $U$ as a disjoint union of open intervals.
For each $x\in U$, we define $\alpha_x=\inf\{\alpha\in\mathbb R:(\alpha,x+\epsilon)\subseteq U, \text{ for some }\epsilon>0\}$ and $\beta_x=\sup\{\beta\in\mathbb R:(\alpha_x,\beta)\subseteq U\}$.

Then $\displaystyle U=\bigcup_{x\in U}(\alpha_x,\beta_x)$ where $\{(\alpha_x,\beta_x):x\in U\}$ is a disjoint family of open intervals.

The intervals appearing in the union are disjoint in the sense that every time $x,y\in U$ with $x<y$, then either $(\alpha_x,\beta_x)=(\alpha_y,\beta_y)$ holds, or $(\alpha_x,\beta_x)\cap(\alpha_y,\beta_y)$ is empty. To see this, suppose $(\alpha_x,\beta_x)\cap(\alpha_y,\beta_y)$ has an element. We claim that $[x,y]\subseteq U$. (For if $x<t<y$ with $t\not\in U$, then $\beta_x\leq t\leq \alpha_y$.)

But if $[x,y]\subseteq U$, then both $\alpha_x<x$ and $\alpha_y<x$, and both $y<\beta_x$ and $y<\beta_y$. Hence $\alpha_x$ and $\alpha_y$ can be expressed as $$\alpha_x=\inf\{\alpha\leq x:(\alpha,x+\epsilon)\subseteq U, \text{ for some }\epsilon>0\},$$ $$\alpha_y=\inf\{\overline\alpha\leq x:(\overline\alpha,y+\overline\epsilon)\subseteq U, \text{ for some }\overline\epsilon>0\},$$ and these are the same; so then also $\beta_x$ and $\beta_y$ are the same.

311411
  • 2,820
  • 6
  • 15
  • 34
P..
  • 14,509
  • 2
  • 29
  • 52
  • 1
    Are you sure the union is countable here? – user10444 Sep 11 '14 at 16:55
  • 3
    @user10444: It is countable yes! If you agree that $\{(\alpha_x,\beta_x):x\in U\}$ is a disjoint family of open intervals then you can see it by choosing $r_x\in \mathbb Q\cap (\alpha_x,\beta_x)$ for all $x\in U$. Then $\{r_x:x\in U\}$ is countable right? Note that the intervals $\{(\alpha_x,\beta_x):x\in U\}$ are not distinct! – P.. Sep 11 '14 at 19:03
  • 3
    Recall, we can "choose" such an $r_{x}\in(\alpha_{x},\beta_{x})$ such that $r_{x}\in\mathbb{Q}$ by the Axiom of Choice and by the density of $\mathbb{Q}$ in $\mathbb{R}$. – Procore Aug 30 '17 at 20:29
15

This proof is an extended version of the nice proof proposed by Stromael and it serves best for beginners who want to understand every detail(that one that for any established mathematician logically seems trivial) of the proof.

$ \textbf{Proof:} $

Let $U \subseteq \mathbb{R}$ be open and let $x \in U$. Then Either $x$ is rational or $x$ is irrational.

Suppose $x$ is rational, then define

\begin{align} I_x = \bigcup\limits_{\substack{I\text{ an open interval} \\ x~\in~I~\subseteq~U}} I,\end{align}

Claim: $I_x$ is interval, $I_x$ is open and $ I_x \subseteq U $

Definition: An interval is a subset $ I \subseteq \mathbb{R}$ such that, for all $ a<c<b$ in $\mathbb{R}$, if $ a,b \in I $ then $ c \in I$.

Now, consider any $ a<c<b $ such that $ a,b \in I_x$. We want to show that $ c \in I_x $.

Denote $I_a $ to be an interval such that $ x \in I_a $ and $ a \in I_a $. In other words $ I_a $ is one of the intervals from the union $ I_x $ that contains $a$. In the same way, let $ I_b $ be the interval such that $ x \in I_b $ and $ b \in I_b $.

  1. $ c=x $: If $c=x$ then by construction of $I_x$, $ c \in I_x$

  2. $ c<x $: If $c<x$ then we have that either $ a<c<x<b $ or $ a<c<b<x $. Since $ x \in I $ for every open interval $I$ of the union $I_x$ (by construction of $I_x$ ), we have that $x \in I_a $ and $ x \in I_b$. Since $ x \in I_a $ then because $ I_a $ is an interval $ c \in I_a$ and hence $ c \in I_x $. And since $ x \in I_b $ then because $ I_b $ is an interval $ c \in I_b $ and hence $ c \in I_x $. Thus, we concluded that $ c \in I_x $.

  3. $ c > x $: If $ c>x $ then we have that either $ a<x<c<b $ or $ x<a<c<b $. Since $ x \in I $ for every open interval $I$ of the union $I_x$ (by construction of $I_x$ ), we have that $x \in I_a $ and $ x \in I_b$. Since $ x \in I_b $ then because $ I_b $ is an interval $ c \in I_b $ and hence $ c \in I_x $. As for the second case, note that since $ x \in I_b$ we have that $ a \in I_b $. But then, because $ I_b $ is an interval we have that $ c \in I_b $ and hence $ c \in I_x$. Hence we concluded that $ c \in I_x $.

This Proves that $ I_x $ is an interval.

$ I_x $ is open because it is union of open sets.

$ I_x \subseteq U $ by construction.

Suppose $x$ is irrational, then by openness of $ U $ there is $\varepsilon > 0$ such that $(x - \varepsilon, x + \varepsilon) \subseteq U$, and by the property of real numbers that for any irrational number there exists a sequence of rational unmbers that converges to that irrational number, there exists rational $y \in (x - \varepsilon, x + \varepsilon) $. Then by construction $ (x - \varepsilon, x + \varepsilon) \subseteq I_y $. Hence $x \in I_y$. So any $x \in U$ is in $I_q$ for some $q \in U \cap \mathbb{Q}$, and so

\begin{align}U \subseteq \bigcup\limits_{q~\in~U \cap~\mathbb{Q}} I_q.\end{align}

But $I_q \subseteq U$ for each $q \in U \cap \mathbb{Q}$; thus

\begin{align}U = \bigcup\limits_{q~\in~U \cap~\mathbb{Q}} I_q, \end{align}

which is a countable union of open intervals.

Now let's show that intervals $ \{I_q \} ~\ q \in U \cap \mathbb{Q} $ are disjoint. Suppose there is $ i, j, \in U \cap \mathbb{Q} $ such that $ I_i \cap I_j \neq \emptyset $ then $ I_i \subseteq I_q $ and $ I_j \subseteq I_q $ for some $ q \in U \cap \mathbb{Q} $

Hence we constructed disjoint intervals $ \{I_q \} ~\ q \in U \cap \mathbb{Q} $ that are enumerated by rational numbers in $U$ and whose union is $U$. Since any subset of rational numbers is countable, $ \{I_q \} ~\ q \in U \cap \mathbb{Q} $ is countable as well. This finishes the proof.

Yia War
  • 15
  • 5
G.T.
  • 440
  • 3
  • 17
  • 1
    What is the guarantee of the statement 'Denote $I_a $ to be an interval such that $ x \in I_a $ '? –  Dec 30 '18 at 14:47
  • Also to me the first part of this answer seemed obscure, I tried to clarify it here https://math.stackexchange.com/questions/3040319/every-open-subset-u-subseteq-mathbbr-is-countable-union-of-disjoint-open-int – Jack J. Jan 17 '19 at 09:28
  • Most probably I am very late for this comment but still I will clarify. By construction $ I_x $ is such that $ x \in $ every $ I $ that forms $ I_x $. Then I say that $ a \in I_x $ this implies that $ a $ must be a member of one of the $ I $s that form $ I_x$. Why? Because if this is not the case then $ a $ is not in $ I_x $. Then I just take and call $ I_a $ one of the $ I $s that contain that $a$. Since $ x \in $ every I $ x $ will be a member of that particular $ I $ that we defined as $ I_a $Let me know if this is wrong and why. – G.T. Sep 05 '20 at 07:15
9

A variant of the usual proof with the equivalence relation, which trades in the ease of constructing the intervals with the ease of proving countability (not that either is hard...):

  1. Define the same equivalence relation, but only on $\mathbb Q \cap U$: $q_1 \sim q_2$ iff $(q_1, q_2) \subset U$ (or $(q_2, q_1) \subset U$, whichever makes sense).
  2. From each equivalency class $C$, produce the open interval $(\inf C, \sup C) \subset U$ (where $\inf C$ is defined to be $-\infty$ in case $C$ is not bounded from below, and $\sup C = \infty$ in case $C$ is not bounded from above).
  3. The amount of equivalence classes is clearly countable, since $\mathbb Q \cap U$ is countable.
Yoni Rozenshein
  • 6,534
  • 22
  • 40
8

Let $U$ be an open subset of $\mathbb{R}$. Let $P$ be the poset consisting of collections $\mathcal{A}$ of disjoint open intervals where we say $\mathcal{A} \le \mathcal{A}'$ if each of the sets in $\mathcal{A}$ is a subset of some open interval in $\mathcal{A}'$. Every chain $C$ in this poset has an upper bound, namely $$\mathcal{B} = \left\{ \bigcup\left\{J \in \bigcup\bigcup C : I \subseteq J \right\}: I \in \bigcup\bigcup C\right\}.$$ Therefore by Zorn's lemma the poset $P$ has a maximal element $\mathcal{M}$. We claim that the union of the intervals in $\mathcal{M}$ is all of $U$. Suppose toward a contradiction that there is a real $x \in U$ that is not contained in any of the intervals in $\mathcal{M}$. Because $U$ is open we can take an open interval $I$ with $x \in I \subseteq U$. Then the set $$\mathcal{M}' = \{J \in \mathcal{M} : J \cap I = \emptyset\} \cup \left\{I \cup \bigcup \{J \in \mathcal{M} : J \cap I \ne \emptyset\}\right\}$$ is a collection of disjoint open intervals and is above $\mathcal{M}$ in the poset $P$, contradicting the maximality of $\mathcal{M}$. It remains to observe that $\mathcal{M}$ is countable, which follows from the fact that its elements contain distinct rational numbers.

Note that the only way in which anything about order (or connectedness) is used is to see that $I \cup \bigcup \{J \in \mathcal{M} : J \cap I \ne \emptyset\}$ is an interval.

Trevor Wilson
  • 16,402
  • 30
  • 66
7

$\mathbb{R}$ with standard topology is second-countable space.

For a second-countable space with a (not necessarily countable) base, any open set can be written as a countable union of basic open set.

Given any base for a second countable space, is every open set the countable union of basic open sets?

Clearly, collection of open intervals is a base for the standard topology. Hence any open set in $\mathbb{R}$ can be written as countable union of open intervals.

If any two of exploited open intervals overlap, merge them. Then we have disjoint union of open intervals, which is still countable.

Guldam
  • 904
  • 7
  • 20
6

Let $G$ be a nonempty open set in $\mathbb{R}$. Write $a\sim b$ if the closed interval $[a, b]$ or $[b, a]$ if $b<a$, lies in $G$.This is an equivalence relation, in particular $a\sim a$ since $\{a\}$ is itself a closed interval. $G$ is therefore the union of disjoint equivalence classes.

Let $C(a)$ be the equivalence class containing $a$. Then $C(a)$ is clearly an interval. Also $C(a)$ is open, for if $k\in C(a)$, then $(k-\epsilon, k+\epsilon)\subseteq G$ for sfficiently small $\epsilon$.

But then $(k-\epsilon, k+\epsilon)\subseteq C(a)$; so $G$ is the union of disjoint intervals. These are at most countable in number by Lindel$\ddot{\rm o}$f's theorem. This completes the proof.

Reference:G. De Barra, Measure theory and Integration, Horwood Publishing.

(http://infoedu.ir/wp-content/uploads/2014/03/MeasureTheoryBook.pdf)

6

The balls with radii $\frac{1}{n}$ and center at a rational number form a basis for the euclidean topology. This family is countable as it is a countable union of countable sets. We have found a countable basis, so we are done.

Asinomás
  • 102,465
  • 18
  • 125
  • 258
  • 6
    Ah, but the intervals are requested to be disjoint, and your intervals aren’t disjoint. – Lubin Oct 08 '16 at 03:11
5

Essentially nothing differs here from the two previous responses which rely principally on the fact that $\mathbb{R} $ is locally connected. However I present here a proof that hopefully will feel accessible to readers with a slightly lower level of topological literary at the cost of appearing cumbersome to an expert.

Consider the connected components of $U$. $U_x \subseteq U$ is defined to be the connected component of U containing $x$ if $U_x$ is the largest connected subset of $U$ which contains $x$. Clearly by definition $U_x=U_v$ if $v \in U_x$. Therefore if $U_a \cap U_b \neq \varnothing$ then $U_a=U_b$. We see that $\{U_x\}_{x\in U}$ is a disjoint collection. Also it should be clear that $\bigcup \limits_{x\in U} U_x = U$.

Now we show that $\forall x$ $U_x$ is open. Let $y\in U_x \subseteq U$. Since $U$ is open there exists $\epsilon>0$ such that $(y-\epsilon, y+\epsilon )\subseteq U$. Sets of real numbers are connected iff they are intervals, singletons or empty. $(y-\epsilon,y+\epsilon)$ an interval hence it is connected. Therefore since $U_y$ is the largest connected subset of $U$ containing $y$ we must have $(y-\epsilon,y+\epsilon)\subseteq U_y =U_x$. This shows that $U_x$ is open for all $x$.

$U_x$ open and connected implies that $U_x$ must be an open interval.

Also $\mathbb{Q}$ dense in $\mathbb{R}$, so $\forall x\in U$, $U_x\cap \mathbb{Q}\neq \varnothing$ and $U_x=U_q$ for some $q\in\mathbb{Q}$. So we can write $\{U_x\}_{x\in U}=\{U_q\}_{q\in S}$ for some $S\subseteq \mathbb{Q}$. $\mathbb{Q}$ is countable so $S$ is at most countable.

In conclusion, we have just shown that the union of the connected components of $U$ is a disjoint union of open intervals that equals $U$ and is at most countable.

Nathan A.S.
  • 66
  • 1
  • 5
5

I hope that this is right as this is a lemma i've thought of and i plan to use in a project due in several days and it somewhat generalizes the question asked:

Suppose that $U$ is a set of intervals in $\mathbb{R}$ (closed, open, semi-closed, etc.). Then there exists a set of disjoint intervals $V$ in $\mathbb{R}$ s.t. $\bigcup_{I\in U}I=\biguplus_{I\in V}I$. If non of the intervals are degenerate or $U$ is countable, then this set can be taken to be countable. And if they were all open, then we could take the segments of $V$ to be open (And also, if $U$ is countable then we won't be needing the Axiom of Choice).

proof: Let us order the elements of $U$: $U=\langle I_\beta\,|\,\beta\leq\alpha\rangle$ where $\alpha$ is the first ordinal of cardinality $|U|$. (If $U$ is countable then this doesn't require AC and from here on will be standard induction with a simple construction in the end for $\omega$).

I'll build $V_\beta=\langle J^\gamma_\beta\,|\,\gamma\leq\beta\rangle$ - a sequence of segments for all $\beta\leq\alpha$ such that every two sets in $V_\beta$ are either disjoint or equal and such that $\displaystyle{\bigcup_{\gamma\leq\beta}I_\gamma=\biguplus_{\gamma\leq\beta}J^\gamma_\beta}$ and $\forall\beta$, $\langle J^\beta_\gamma\rangle_{\gamma\geq\beta}$ is a non-descending sequence of sets ,by means of transfinite induction.

For $V_0$ take, $V_0=\langle I_0\rangle$. Suppose that we have built the required $V_\gamma$, $\gamma<\beta$ for some $\beta\leq\alpha$, then we will build $V_\beta$ in the following way: $\forall\gamma<\beta$, denote $\widetilde{J}_\gamma$=$\bigcup_{\gamma\leq\delta<\beta}J_\delta^\gamma$-still segments (non-decreasing sequence). If $I_\beta$ is disjoint of all $\widetilde{J}_\gamma$, taking $V_\beta\!=\!\langle \widetilde{J}_\gamma\,|\,\gamma<\beta\rangle\cup\{(\beta,I_\beta)\}$ would give us a sequence $\langle V_\gamma\,|\,\gamma\leq\beta\rangle$ satisfying the required conditions of it (the only non trivial thing is that pairs of $\widetilde{J}_\gamma$ are either disjoint of each other or are equal, but that is also quite trivial since if the contrary would have occurred, then $\exists\gamma_1<\gamma_2<\beta$ s.t. $\widetilde{J}_{\gamma_1}\neq\widetilde{J}_{\gamma_2}$ and $\widetilde{J}_{\gamma_1}\cap\widetilde{J}_{\gamma_2}\neq\emptyset$, but then, $\exists \beta>\delta_1\geq\gamma_1, \beta>\delta_2\geq\gamma_2$ s.t. $J^{\gamma_1}_{\delta_1}\cap J^{\gamma_2}_{\delta_2}\neq\emptyset$, meaning that either $J^{\gamma_1}_{\delta_2}= J^{\gamma_2}_{\delta_2}$ or $J^{\gamma_1}_{\delta_1}= J^{\gamma_2}_{\delta_1}$ thus, $\forall\beta>\epsilon\geq\delta_1,\delta_2$, $J^{\gamma_1}_{\epsilon}= J^{\gamma_2}_{\epsilon}$ and since we are talking here about non-decreasing sequences, this will contradict $\widetilde{J}_{\gamma_1}\neq\widetilde{J}_{\gamma_2}$). And if $I_\beta$ isn't disjoint of all $\widetilde{J}_\gamma$, Then we can take $J_\beta^\gamma=\widetilde{J}_\gamma$ for all $\gamma<\beta$ that don't intersect with $I_\beta$ and $J_\beta^\gamma=\bigcup_{\delta<\beta\text{ s.t. }\widetilde{J}_\delta\cap I_\beta\neq\emptyset}{\widetilde{J}_\delta}\cup I_\beta$ - segment for all of the other $\gamma\leq\beta$. Then again from the same arguments, $\langle V_\gamma\,|\,\gamma\leq\beta\rangle$ would satisfy the required conditions.

Finally, we can take $V=\{J_\alpha^\beta\,|\,\beta\leq\alpha\}$ to get what we wanted in the first place. And obviously, if our segments were all non-degenerate to begin with, from the way we constructed our set, all of the segments in $V$ will be non-degenerate (and thus of positive measure), but they are disjoint and so there is only a countable number of them. And if the segments in $U$ were all open, then obviously, so will the segments in $V$.$\square$

peter
  • 90
  • 1
  • 5
4

The following is certainly not the quickest approach to a proof, but when this question was first posed to me in class, my first intuition was to use some elementary graph theory:


Let $U$ be an open set of $\mathbb{R}$. As we know, $\mathbb{R}$ has a countable basis $\mathcal{B}$ comprised of connected open sets and so we may write $U=\bigcup_{n\in I} U_n$, where for each $n$ we have $U_n\in\mathcal{B}$ and $I$ is some countable index set.

Let $G$ be the intersection graph of $\{U_n\}$. That is to say, the vertex set of $G$ is simply $\{U_n\}$ and there is an edge between $U_i$ and $U_j$ iff they have nonempty intersection. It's easy to convince yourself that:

  • This graph must have countably many graphically-connected components (otherwise we'd have uncountably many vertices which is impossible).
  • The intersection graph of $A\subseteq\{U_n\}$ is graphically-connected iff for any two $V,W\in A$ there is a sequence $V=U_{n_1},U_{n_2},\ldots,U_{n_k}=W$ such that $U_{n_i}\cap U_{n_{i+1}}\neq\varnothing$.
  • The union $\bigcup A$ is a connected set of $\mathbb{R}$ whenever the intersection graph of $A$ is graphically-connected.

Thus, when we take the union of all the vertices within a graphically-connected component, for every component, we obtain countably-many connected open sets. The union of these sets is of course $U$ itself. Since the connected open sets of $\mathbb{R}$ are intervals (including rays), we're done.


Side Note: This would also work in $\mathbb{R}^n$ or in general, any topological space $X$ that has a countable basis comprised of connected sets. Well, so long as we replace countable union of disjoint open intervals with countable union of disjoint open connected sets.

Christian Bueno
  • 1,169
  • 7
  • 20
4

The balls with radii $\frac{1}{n}$ and center at a rational number form a basis for the euclidean topology. This family is countable since $\mathbb N \times \mathbb Q \equiv \mathbb N$ and we have a countable basis $(B_\lambda)_{ \, \lambda \in \mathbb N \times \mathbb Q}$ of open intervals for $\mathbb R$.

Let $U$ be a nonempty open set in $\mathbb R$; we can express it as countable union of open balls from $(B_\lambda)$. Also, $\tag 1 \text{ }$ $\quad$ If two open intervals have a nonempty intersection, then their union is also an open interval.

So if $U$ is a finite union of the $B_\lambda$, it is an easy matter to combine the $B_\lambda$, if necessary, and writing $U$ as a disjoint union of a finite number of open intervals. So assume, WLOG, that

$\tag 2 U = \bigcup_{\, n \in \mathbb N \,} B_n$.

We define a relation on our (new) index set $\mathbb N$ with $m\sim n$ if $B_m \cap B_n \ne \emptyset$ or there is is a finite 'nonempty intersection $B\text{-}$chain' connecting $B_m$ with $B_n$. It is easy to see that this partitions $\mathbb N$ and that taking the corresponding unions of the $B_n$ over an index $\lambda \text{-}$ block gives a partition of $U$. Also, using (1), we can show that we have also expressed $U$ as a countable union of disjoint open intervals.

CopyPasteIt
  • 10,521
  • 1
  • 17
  • 43
4

The proof that every open set is a disjoint union of countably many open intervals relies on three facts:

  • $\Bbb R$ is locally-connected
  • $\Bbb R$ is ccc
  • The open connected sets in $\Bbb R$ are open intervals

Let $U\subseteq \Bbb R$ be open. Then there is a collection of disjoint, open, connected sets $\{G_\alpha\}_{\alpha\in A}$ such that $U=\bigcup_{\alpha\in A} G_\alpha$. Since $\Bbb R$ is ccc, the collection $\{G_\alpha\}$ is at most countable. Since the open connected sets $\Bbb R$ are open intervals, $\{G_\alpha\}$ is a countable collection of disjoint, open intervals.

The first two facts allow us to see some generalizations. Namely any open set in a locally-connected, ccc space is a countable disjoint union of connected open sets. This applies to any Euclidean space. Although open connected subsets of Euclidean space are more complicated than open intervals, they are still relatively well-behaved.

3

More of a question than answer. I am a chemist turned pharmacist, who wishes to have studied mathematics. I am trying to work through Rudin's Principles of Mathematical Analysis. I envy you all who are involved in math for a career.

Can someone give me feedback on my attempt at a proof in $\mathbb R? Completely novice and not at all pretty, but is it sound?

segment := open interval in R.

Lemma: disjoint segments in R are separated (proof not shown).

It follows from the lemma that an open connected subset of R cannot be the union of disjoint segments.

Let E be an open subset of R. Since R is separable by Rudin prob 2.22, there exists a subset, D, of R that is countable and dense in R. Assume E is connected, which includes E=R, then E is the union of an at most countable collection of open segments, containing only E.

Suppose E is separated. Then E is the union of a collection of disjoint segments, including the possibility of segments unbounded above or below.

If the collection is finite, then it is at most countable.

Assume the collection of segments is infinite. Because D is dense in R and E is contained in R, every open subset of E contains a point of D. Then each of the infinitely many disjoint segments contains a unique point of D. Since D is countable, a one-to-one correspondence between a unique point of D and the segment is established. This implies that there are countably many disjoint segments in the collection.

Therefore E is the union of finitely many or countably many , hence at most countably many, disjoint segments.

amWhy
  • 204,278
  • 154
  • 264
  • 488
RJM
  • 1,097
  • 9
  • 21
  • 1
    `More of a question than answer.` I suppose this would be better posted as a question than as an answer, then. `Suppose E is separated. Then E is the union of a collection of disjoint segments.` It's not obvious (to me, at least) where this follows from in the given context. – dxiv Oct 09 '16 at 05:43
  • 1
    I know. Just learning how things work on this site. Which I think is quite amazing. – RJM Oct 09 '16 at 05:48
  • @dxiv Since E separated (or not connected), would it have to at least be the union of 2 separated sets. Since it is open, they have to be open subsets, which implies open interval in R1. Thank you! – RJM Oct 09 '16 at 05:53
  • @dxiv i will not keep bugging you. Any tips on how to become mathematician-like on one's own in essentially matgematica isolation? – RJM Oct 09 '16 at 06:00
  • IMHO that latter `implies open interval in R1` is a part that needs to be proved in this context. See for example [Intervals are connected and the only connected sets in R](http://math.stackexchange.com/questions/239063/intervals-are-connected-and-the-only-connected-sets-in-mathbbr). Anyway, as I said, yours appears to be better suited as a question than as an answer. P.S. Keep plugging away until it all starts making sense ;-) – dxiv Oct 09 '16 at 06:00
  • Thank you. I really appreciate your help. – RJM Oct 09 '16 at 06:03